0% found this document useful (0 votes)
76 views4 pages

CAT

Uploaded by

aroridouglas880
Copyright
© © All Rights Reserved
We take content rights seriously. If you suspect this is your content, claim it here.
Available Formats
Download as PDF, TXT or read online on Scribd
0% found this document useful (0 votes)
76 views4 pages

CAT

Uploaded by

aroridouglas880
Copyright
© © All Rights Reserved
We take content rights seriously. If you suspect this is your content, claim it here.
Available Formats
Download as PDF, TXT or read online on Scribd
You are on page 1/ 4

MMA 208-INTRODUCTION TO ANALYSIS

SIT IN CAT II SOLUTIONS

(a) Consider the following functions defined on R,

f (x) = tan x, g(x) = e−x , h(x) = x2 .

Give the range of each of the above functions. [3 marks]

Solution.

ˆ Range(f ) = R or (−∞, ∞)
ˆ Range(g) = (0, ∞)
ˆ Range(h) = [0, ∞)

Solution.

(b) Let S = [−10, −1) ∪ (0, 1) ∪ (1.5, 3) ∪ {3}. Show that S is neither closed nor open.

Observe that

ˆ Int(S) = (−10, −1) ∪ (0, 1) ∪ (1.5, 3) ̸= S. ⇒ S is not open.


ˆ S̄ = [−10, −1] ∪ [0, 1] ∪ [1.5, 3] ̸= S. ⇒ S is not closed.

[3 marks]

(c) Show that the set [−1, 1] is not a neighbourhood of the points −1 and 1. [2 marks]

Solution. For each ε > 0, the open interval (−1 − ε, −1 + ε) contains an


element −1 − 2ε ̸∈ [−1, 1] and (1 − ε, 1 + ε) contains an element 1 + 2ε ̸∈ [−1, 1].
Therefore, no open interval that contains −1 or 1 is fully contained in [−1, 1].

(d) Show that lim 9x − 3 = 15. [4 marks].


x→2

Solution. Let f (x) = 9x − 3, c = 2 and l = 15. We show that, ∀ε > 0, ∃δ > 0


such that |f (x) − l| < ε, whenever |x − c| < δ.
Let ϵ > 0 be given. Then |f (x) − l| < ε. Observe that,

|f (x) − l| = |(9x − 3) − 15| = |9x − 18| = |9(x − 2)| = 9|x − 2| < ε


ε
⇒|x − 2| < = δ.
2

1
(e) Show that the set of all integers (Z) is not a field. [2 marks]

Solution. Since the only nonzero integers with a multiplicative inverse are 1
and −1.

(f) Let x, y ∈ R. Prove that, if |x| < δ, δ > 0, then −δ < x < δ. [2 marks]

Solution. By definition, |x| = max{−x, x}. Thus, −x < δ and x < δ. If


we multiply both sides of the former by −1 we obtain that −δ < x. Therefore,
−δ < x and x < δ obtains −δ < x < δ.

(g) Write any 4 nontrivial partitions of the set A = {1, 2, 3, 4, 5, 6, 7, 8}. [4 marks]

[Not part of the solution: There are many partitions for this set, in fact the
total number of partitions for this set is 4140 so we will just list a few random
partitions:
Solution.

ˆ P1 = {{1, 2, 3}, {4, 5}, {6, 7, 8}}


ˆ P2 = {{1, 2}, {3, 4}, {5, 6}, {7, 8}}
ˆ P3 = {{1, 2, 3, 4, 5}, {6, 7, 8}}
ˆ P4 = {{1, 2, 3}, {4, 5, 6, 7, 8}}
ˆ P5 = {{1, 2, 3, 4, 5, 6, 7}, {8}}

(h) Define a relation R on a set X = {a, b, c, d} defined by

R = {(a, b), (b, a), (c, a), (d, a)}.

Determine whether or not R defines a function. [2 marks]

Solution. Since every element of X is assigned to one and only one image in
X, we can say that R is a function.

(i) Let E denote the set of all even numbers and f : E → Z defined by
n+2
f (n) = .
2
Show that f is both one to one and onto. [4 marks]

2
Solution. To show that f is 1-1, Let a, b ∈ E be such that f (a) = f (b). Then

a+2 b+2
f (a) = = = f (b), ⇔ a + 2 = b + 2, ⇔ a = b
2 2
Therefore, f is 1-1.
To show f is onto we need to find an inverse of every integer n. Suppose that k
is such inverse of n, then f (k) = n. It suffices to show that k is even.

k+2
f (k) = = n, ⇔ k + 2 = 2n, ⇔ k = 2n − 2 = 2(n − 1).
2
For any integer n, the number k = 2(n − 1) is an even integer.

(j) Show that the open interval (a, b), a, b ∈ R is an open set. [4 marks]

Solution. We show that (a, b) is a neighborhood of each of its points. Let


x ∈ (a, b) be an arbitrary point of (a, b) and let 0 < ε < min{x − a, b − x}. Then
x ∈ (x − ε, x + ε) ⊂ (a, b). Therefore, (a, b) is a neighborhood of each of its points.

(k) Let Consider the function



1,


0<x<1
x, 1<x<2
f (x) =


 −1, 2<x<3
x ≥ 3.

0,

(i) Sketch the graph of f (x). [3 marks]

Solution.
4

0
−1 0 1 2 3 4 5
−1

−2

(ii) Determine whether or not, lim f (x) exists. [3 marks]


x→1

3
Solution. Observe that

lim f (x) = 1 = lim+ f (x)


x→1− x→1

Therefore since the two limits exist and are equal, limx→1 f (x) exists and is
equal to 1.

(iii) Discuss the continuity and discontinuity of f (x). [3 marks]

Solution. The function f is continuous everywhere on the interval (0, ∞)


except at x = 1, 2, 3.
At x = 1, f has a removable discontinuity, at x = 2 and 3 f has a disconti-
nuity of the first kind.

You might also like

pFad - Phonifier reborn

Pfad - The Proxy pFad of © 2024 Garber Painting. All rights reserved.

Note: This service is not intended for secure transactions such as banking, social media, email, or purchasing. Use at your own risk. We assume no liability whatsoever for broken pages.


Alternative Proxies:

Alternative Proxy

pFad Proxy

pFad v3 Proxy

pFad v4 Proxy